We have your learning goals covered with Spring and Summer courses available. Enroll today!

G
Topic
First Poster
Last Poster
k a My Retirement & New Leadership at AoPS
rrusczyk   1571
N Mar 26, 2025 by SmartGroot
I write today to announce my retirement as CEO from Art of Problem Solving. When I founded AoPS 22 years ago, I never imagined that we would reach so many students and families, or that we would find so many channels through which we discover, inspire, and train the great problem solvers of the next generation. I am very proud of all we have accomplished and I’m thankful for the many supporters who provided inspiration and encouragement along the way. I'm particularly grateful to all of the wonderful members of the AoPS Community!

I’m delighted to introduce our new leaders - Ben Kornell and Andrew Sutherland. Ben has extensive experience in education and edtech prior to joining AoPS as my successor as CEO, including starting like I did as a classroom teacher. He has a deep understanding of the value of our work because he’s an AoPS parent! Meanwhile, Andrew and I have common roots as founders of education companies; he launched Quizlet at age 15! His journey from founder to MIT to technology and product leader as our Chief Product Officer traces a pathway many of our students will follow in the years to come.

Thank you again for your support for Art of Problem Solving and we look forward to working with millions more wonderful problem solvers in the years to come.

And special thanks to all of the amazing AoPS team members who have helped build AoPS. We’ve come a long way from here:IMAGE
1571 replies
rrusczyk
Mar 24, 2025
SmartGroot
Mar 26, 2025
k a March Highlights and 2025 AoPS Online Class Information
jlacosta   0
Mar 2, 2025
March is the month for State MATHCOUNTS competitions! Kudos to everyone who participated in their local chapter competitions and best of luck to all going to State! Join us on March 11th for a Math Jam devoted to our favorite Chapter competition problems! Are you interested in training for MATHCOUNTS? Be sure to check out our AMC 8/MATHCOUNTS Basics and Advanced courses.

Are you ready to level up with Olympiad training? Registration is open with early bird pricing available for our WOOT programs: MathWOOT (Levels 1 and 2), CodeWOOT, PhysicsWOOT, and ChemWOOT. What is WOOT? WOOT stands for Worldwide Online Olympiad Training and is a 7-month high school math Olympiad preparation and testing program that brings together many of the best students from around the world to learn Olympiad problem solving skills. Classes begin in September!

Do you have plans this summer? There are so many options to fit your schedule and goals whether attending a summer camp or taking online classes, it can be a great break from the routine of the school year. Check out our summer courses at AoPS Online, or if you want a math or language arts class that doesn’t have homework, but is an enriching summer experience, our AoPS Virtual Campus summer camps may be just the ticket! We are expanding our locations for our AoPS Academies across the country with 15 locations so far and new campuses opening in Saratoga CA, Johns Creek GA, and the Upper West Side NY. Check out this page for summer camp information.

Be sure to mark your calendars for the following events:
[list][*]March 5th (Wednesday), 4:30pm PT/7:30pm ET, HCSSiM Math Jam 2025. Amber Verser, Assistant Director of the Hampshire College Summer Studies in Mathematics, will host an information session about HCSSiM, a summer program for high school students.
[*]March 6th (Thursday), 4:00pm PT/7:00pm ET, Free Webinar on Math Competitions from elementary through high school. Join us for an enlightening session that demystifies the world of math competitions and helps you make informed decisions about your contest journey.
[*]March 11th (Tuesday), 4:30pm PT/7:30pm ET, 2025 MATHCOUNTS Chapter Discussion MATH JAM. AoPS instructors will discuss some of their favorite problems from the MATHCOUNTS Chapter Competition. All are welcome!
[*]March 13th (Thursday), 4:00pm PT/7:00pm ET, Free Webinar about Summer Camps at the Virtual Campus. Transform your summer into an unforgettable learning adventure! From elementary through high school, we offer dynamic summer camps featuring topics in mathematics, language arts, and competition preparation - all designed to fit your schedule and ignite your passion for learning.[/list]
Our full course list for upcoming classes is below:
All classes run 7:30pm-8:45pm ET/4:30pm - 5:45pm PT unless otherwise noted.

Introductory: Grades 5-10

Prealgebra 1 Self-Paced

Prealgebra 1
Sunday, Mar 2 - Jun 22
Friday, Mar 28 - Jul 18
Sunday, Apr 13 - Aug 10
Tuesday, May 13 - Aug 26
Thursday, May 29 - Sep 11
Sunday, Jun 15 - Oct 12
Monday, Jun 30 - Oct 20
Wednesday, Jul 16 - Oct 29

Prealgebra 2 Self-Paced

Prealgebra 2
Tuesday, Mar 25 - Jul 8
Sunday, Apr 13 - Aug 10
Wednesday, May 7 - Aug 20
Monday, Jun 2 - Sep 22
Sunday, Jun 29 - Oct 26
Friday, Jul 25 - Nov 21


Introduction to Algebra A Self-Paced

Introduction to Algebra A
Sunday, Mar 23 - Jul 20
Monday, Apr 7 - Jul 28
Sunday, May 11 - Sep 14 (1:00 - 2:30 pm ET/10:00 - 11:30 am PT)
Wednesday, May 14 - Aug 27
Friday, May 30 - Sep 26
Monday, Jun 2 - Sep 22
Sunday, Jun 15 - Oct 12
Thursday, Jun 26 - Oct 9
Tuesday, Jul 15 - Oct 28

Introduction to Counting & Probability Self-Paced

Introduction to Counting & Probability
Sunday, Mar 16 - Jun 8
Wednesday, Apr 16 - Jul 2
Thursday, May 15 - Jul 31
Sunday, Jun 1 - Aug 24
Thursday, Jun 12 - Aug 28
Wednesday, Jul 9 - Sep 24
Sunday, Jul 27 - Oct 19

Introduction to Number Theory
Monday, Mar 17 - Jun 9
Thursday, Apr 17 - Jul 3
Friday, May 9 - Aug 1
Wednesday, May 21 - Aug 6
Monday, Jun 9 - Aug 25
Sunday, Jun 15 - Sep 14
Tuesday, Jul 15 - Sep 30

Introduction to Algebra B Self-Paced

Introduction to Algebra B
Sunday, Mar 2 - Jun 22
Wednesday, Apr 16 - Jul 30
Tuesday, May 6 - Aug 19
Wednesday, Jun 4 - Sep 17
Sunday, Jun 22 - Oct 19
Friday, Jul 18 - Nov 14

Introduction to Geometry
Tuesday, Mar 4 - Aug 12
Sunday, Mar 23 - Sep 21
Wednesday, Apr 23 - Oct 1
Sunday, May 11 - Nov 9
Tuesday, May 20 - Oct 28
Monday, Jun 16 - Dec 8
Friday, Jun 20 - Jan 9
Sunday, Jun 29 - Jan 11
Monday, Jul 14 - Jan 19

Intermediate: Grades 8-12

Intermediate Algebra
Sunday, Mar 16 - Sep 14
Tuesday, Mar 25 - Sep 2
Monday, Apr 21 - Oct 13
Sunday, Jun 1 - Nov 23
Tuesday, Jun 10 - Nov 18
Wednesday, Jun 25 - Dec 10
Sunday, Jul 13 - Jan 18
Thursday, Jul 24 - Jan 22

Intermediate Counting & Probability
Sunday, Mar 23 - Aug 3
Wednesday, May 21 - Sep 17
Sunday, Jun 22 - Nov 2

Intermediate Number Theory
Friday, Apr 11 - Jun 27
Sunday, Jun 1 - Aug 24
Wednesday, Jun 18 - Sep 3

Precalculus
Sunday, Mar 16 - Aug 24
Wednesday, Apr 9 - Sep 3
Friday, May 16 - Oct 24
Sunday, Jun 1 - Nov 9
Monday, Jun 30 - Dec 8

Advanced: Grades 9-12

Olympiad Geometry
Wednesday, Mar 5 - May 21
Tuesday, Jun 10 - Aug 26

Calculus
Sunday, Mar 30 - Oct 5
Tuesday, May 27 - Nov 11
Wednesday, Jun 25 - Dec 17

Group Theory
Thursday, Jun 12 - Sep 11

Contest Preparation: Grades 6-12

MATHCOUNTS/AMC 8 Basics
Sunday, Mar 23 - Jun 15
Wednesday, Apr 16 - Jul 2
Friday, May 23 - Aug 15
Monday, Jun 2 - Aug 18
Thursday, Jun 12 - Aug 28
Sunday, Jun 22 - Sep 21
Tues & Thurs, Jul 8 - Aug 14 (meets twice a week!)

MATHCOUNTS/AMC 8 Advanced
Friday, Apr 11 - Jun 27
Sunday, May 11 - Aug 10
Tuesday, May 27 - Aug 12
Wednesday, Jun 11 - Aug 27
Sunday, Jun 22 - Sep 21
Tues & Thurs, Jul 8 - Aug 14 (meets twice a week!)

AMC 10 Problem Series
Tuesday, Mar 4 - May 20
Monday, Mar 31 - Jun 23
Friday, May 9 - Aug 1
Sunday, Jun 1 - Aug 24
Thursday, Jun 12 - Aug 28
Tuesday, Jun 17 - Sep 2
Sunday, Jun 22 - Sep 21 (1:00 - 2:30 pm ET/10:00 - 11:30 am PT)
Monday, Jun 23 - Sep 15
Tues & Thurs, Jul 8 - Aug 14 (meets twice a week!)

AMC 10 Final Fives
Sunday, May 11 - Jun 8
Tuesday, May 27 - Jun 17
Monday, Jun 30 - Jul 21

AMC 12 Problem Series
Tuesday, May 27 - Aug 12
Thursday, Jun 12 - Aug 28
Sunday, Jun 22 - Sep 21
Wednesday, Aug 6 - Oct 22

AMC 12 Final Fives
Sunday, May 18 - Jun 15

F=ma Problem Series
Wednesday, Jun 11 - Aug 27

WOOT Programs
Visit the pages linked for full schedule details for each of these programs!


MathWOOT Level 1
MathWOOT Level 2
ChemWOOT
CodeWOOT
PhysicsWOOT

Programming

Introduction to Programming with Python
Monday, Mar 24 - Jun 16
Thursday, May 22 - Aug 7
Sunday, Jun 15 - Sep 14 (1:00 - 2:30 pm ET/10:00 - 11:30 am PT)
Tuesday, Jun 17 - Sep 2
Monday, Jun 30 - Sep 22

Intermediate Programming with Python
Sunday, Jun 1 - Aug 24
Monday, Jun 30 - Sep 22

USACO Bronze Problem Series
Tuesday, May 13 - Jul 29
Sunday, Jun 22 - Sep 1

Physics

Introduction to Physics
Sunday, Mar 30 - Jun 22
Wednesday, May 21 - Aug 6
Sunday, Jun 15 - Sep 14
Monday, Jun 23 - Sep 15

Physics 1: Mechanics
Tuesday, Mar 25 - Sep 2
Thursday, May 22 - Oct 30
Monday, Jun 23 - Dec 15

Relativity
Sat & Sun, Apr 26 - Apr 27 (4:00 - 7:00 pm ET/1:00 - 4:00pm PT)
Mon, Tue, Wed & Thurs, Jun 23 - Jun 26 (meets every day of the week!)
0 replies
jlacosta
Mar 2, 2025
0 replies
Counting Numbers
steven_zhang123   0
a minute ago
Source: China TST 2001 Quiz 8 P3
Let the decimal representations of numbers $A$ and $B$ be given as: $A = 0.a_1a_2\cdots a_k > 0$, $B = 0.b_1b_2\cdots b_k > 0$ (where $a_k, b_k$ can be 0), and let $S$ be the count of numbers $0.c_1c_2\cdots c_k$ such that $0.c_1c_2\cdots c_k < A$ and $0.c_kc_{k-1}\cdots c_1 < B$ ($c_k, c_1$ can also be 0). (Here, $0.c_1c_2\cdots c_r (c_r \neq 0)$ is considered the same as $0.c_1c_2\cdots c_r0\cdots0$).

Prove: $\left| S - 10^k AB \right| \leq 9k.$
0 replies
steven_zhang123
a minute ago
0 replies
Perfect Numbers
steven_zhang123   0
4 minutes ago
Source: China TST 2001 Quiz 8 P2
If the sum of all positive divisors (including itself) of a positive integer $n$ is $2n$, then $n$ is called a perfect number. For example, the sum of the positive divisors of 6 is $1 + 2 + 3 + 6 = 2 \times 6$, hence 6 is a perfect number.
Prove: There does not exist a perfect number of the form $p^a q^b r^c$, where $a, b, c$ are positive integers, and $p, q, r$ are odd primes.
0 replies
steven_zhang123
4 minutes ago
0 replies
Roots of unity
steven_zhang123   0
5 minutes ago
Source: China TST 2001 Quiz 8 P1
Let $k, n$ be positive integers, and let $\alpha_1, \alpha_2, \ldots, \alpha_n$ all be $k$-th roots of unity, satisfying:
\[
\alpha_1^j + \alpha_2^j + \cdots + \alpha_n^j = 0 \quad \text{for any } j (0 < j < k).
\]Prove that among $\alpha_1, \alpha_2, \ldots, \alpha_n$, each $k$-th root of unity appears the same number of times.
0 replies
steven_zhang123
5 minutes ago
0 replies
Graph Theory Test in China TST (space stations again)
steven_zhang123   0
8 minutes ago
Source: China TST 2001 Quiz 7 P3
MO Space City plans to construct $n$ space stations, with a unidirectional pipeline connecting every pair of stations. A station directly reachable from station P without passing through any other station is called a directly reachable station of P. The number of stations jointly directly reachable by the station pair $\{P, Q\}$ is to be examined. The plan requires that all station pairs have the same number of jointly directly reachable stations.

(1) Calculate the number of unidirectional cyclic triangles in the space city constructed according to this requirement. (If there are unidirectional pipelines among three space stations A, B, C forming $A \rightarrow B \rightarrow C \rightarrow A$, then triangle ABC is called a unidirectional cyclic triangle.)

(2) Can a space city with $n$ stations meeting the above planning requirements be constructed for infinitely many integers $n \geq 3$?
0 replies
steven_zhang123
8 minutes ago
0 replies
No more topics!
A_2, B_2,C_2 cannot all lie strictly inside the circumcircle of triangle ABC
parmenides51   25
N Jan 19, 2025 by Scilyse
Source: IMO 2019 SL G4
Let $P$ be a point inside triangle $ABC$. Let $AP$ meet $BC$ at $A_1$, let $BP$ meet $CA$ at $B_1$, and let $CP$ meet $AB$ at $C_1$. Let $A_2$ be the point such that $A_1$ is the midpoint of $PA_2$, let $B_2$ be the point such that $B_1$ is the midpoint of $PB_2$, and let $C_2$ be the point such that $C_1$ is the midpoint of $PC_2$. Prove that points $A_2, B_2$, and $C_2$ cannot all lie strictly inside the circumcircle of triangle $ABC$.

(Australia)
25 replies
parmenides51
Sep 22, 2020
Scilyse
Jan 19, 2025
A_2, B_2,C_2 cannot all lie strictly inside the circumcircle of triangle ABC
G H J
Source: IMO 2019 SL G4
The post below has been deleted. Click to close.
This post has been deleted. Click here to see post.
parmenides51
30628 posts
#1 • 2 Y
Y by Ya_pank, ImSh95
Let $P$ be a point inside triangle $ABC$. Let $AP$ meet $BC$ at $A_1$, let $BP$ meet $CA$ at $B_1$, and let $CP$ meet $AB$ at $C_1$. Let $A_2$ be the point such that $A_1$ is the midpoint of $PA_2$, let $B_2$ be the point such that $B_1$ is the midpoint of $PB_2$, and let $C_2$ be the point such that $C_1$ is the midpoint of $PC_2$. Prove that points $A_2, B_2$, and $C_2$ cannot all lie strictly inside the circumcircle of triangle $ABC$.

(Australia)
Z K Y
The post below has been deleted. Click to close.
This post has been deleted. Click here to see post.
khina
993 posts
#2 • 28 Y
Y by magicarrow, amar_04, ilovepizza2020, OlympusHero, k12byda5h, DVDthe1st, Limerent, TheUltimate123, itslumi, Imayormaynotknowcalculus, Ya_pank, IAmTheHazard, HamstPan38825, kamatadu, CyclicISLscelesTrapezoid, ImSh95, caicasso, Aryan-23, crazyeyemoody907, michaelwenquan, ApraTrip, rjiangbz, awesomeming327., v4913, sabkx, pomodor_ap, Math_legendno12, Funcshun840
i am overjoyed that this did not make the IMO.
Z K Y
The post below has been deleted. Click to close.
This post has been deleted. Click here to see post.
The_Turtle
254 posts
#3 • 21 Y
Y by spartacle, mira74, magicarrow, Atpar, aa1024, khina, jlamslam, Wizard_32, Kanep, franchester, tigerzhang, IAmTheHazard, Tafi_ak, ImSh95, crazyeyemoody907, sabkx, jrsbr, CyclicISLscelesTrapezoid, Rijul saini, Rounak_iitr, Funcshun840
A rather strange solution

Diagram
Z K Y
The post below has been deleted. Click to close.
This post has been deleted. Click here to see post.
v_Enhance
6870 posts
#4 • 11 Y
Y by Mathematicsislovely, Atpar, Math_olympics, v4913, Soundricio, HamstPan38825, ImSh95, crazyeyemoody907, sabkx, Ibrahim_K, Funcshun840
Let $P = (x:y:z)$ with $x,y,z > 0$ so $A_1 = (0:y:z)$. Therefore, \begin{align*} 	A_2 &= \left( -x(y+z) : 2y(x+y+z) - (y+z)y : 2(x+y+z)-(y+z)z \right) \\ 	&= \left( -x(y+z) : y(2x+y+z) : z(2x+y+z)  \right). \end{align*}Claim: The point $A_2$ lies inside $(ABC)$ if and only if \[ 0 < a^2 \cdot yz(2x+y+z) - b^2 \cdot xz(y+z) - c^2xy(y+z). \]Proof. The condition we want is that \begin{align*} 	0 &< -\operatorname{Pow}(A_2, (ABC)) \\ 	&= a^2 \cdot y(2x+y+z) \cdot z(2x+y+z) \\ 		&\qquad - b^2x(y+z) \cdot z(2x+y+z) - c^2x(y+z) \cdot y(2x+y+z) \\ 	&= (2x+y+z) \left[ a^2 \cdot yz(2x+y+z) 		- b^2 \cdot xz(y+z) 		- c^2 \cdot xy(y+z) \right]. \end{align*}From $2x+y+z>0$ we conclude. $\blacksquare$
If all three points lie inside the circle, we sum cyclically now to get a contradiction.
Z K Y
The post below has been deleted. Click to close.
This post has been deleted. Click here to see post.
jj_ca888
2726 posts
#5 • 9 Y
Y by amar_04, Imayormaynotknowcalculus, magicarrow, k12byda5h, CHLORG1, ImSh95, michaelwenquan, jhi1234123, Funcshun840
this is disgusting
Z K Y
The post below has been deleted. Click to close.
This post has been deleted. Click here to see post.
MarkBcc168
1594 posts
#6 • 18 Y
Y by Gaussian_cyber, amar_04, jj_ca888, OlyMan, magicarrow, Atpar, OlympusHero, Eliot, p_square, Rg230403, Ya_pank, Point_Dexter, ImSh95, michaelwenquan, sabkx, jrsbr, CyclicISLscelesTrapezoid, Funcshun840
Comment: This actually has some nice synthetic solutions, but sadly is too easy to bary bash. The first solution is an elementary solution that I found after the exam, while the second one was actually the way I solved it.
Solution 1 (Elementary): The idea is the following claim.

Claim: Suppose that $A_2$ lie on $\odot(ABC)$. Then the isogonal conjugate $Q$ of $P$ w.r.t. $\triangle ABC$ lie on $\odot(AO)$, where $O$ is the circumcenter of $\triangle ABC$.

Proof 1 (Similar Triangles). Extend $AQ$ to meet $\odot(ABC)$ at $X$. Notice that $\triangle BA_2A_1\sim\triangle AXB$. Therefore if $K$ is the reflection of $A_2$ across $B$, we get $KP\parallel BA_1$ so $\triangle KA_2P\sim\triangle AXB$. However,
$$\measuredangle QBA = \measuredangle CBP = \measuredangle KBP\implies \triangle KA_2P\cup B\sim\triangle AXB\cup Q$$hence $Q$ is the midpoint of $AX$, done. $\blacksquare$

Proof 2 (Harmonic). Again, let $X=AQ\cap\odot(ABC)$. Notice that lines $\{BX,B{\infty}_{AP}\}$ and $\{BA_2, B{\infty}_{AQ}\}$ are isogonal w.r.t. $\angle ABC$. Thus by reflecting across angle bisector,
$$B(P,A_2;A_1,{\infty}_{AP}) = B(Q,{\infty}_{AQ};A,X)$$thus $Q$ is the midpoint of $AX$, done. $\blacksquare$

Back to the main problem. Let $Q$ be the isogonal conjugate of $P$ w.r.t. $\triangle ABC$. Suppose that $A_2,B_2,C_2$ all lie inside $\odot(ABC)$. Let $A_2'=AP\cap\odot(ABC)$ and $P'$ be the reflection of $P$ across $A_1$. Observe that $\{P,A_2,A_2'\}$ are colinear in this order so $\{A,P',P\}$ are colinear in this order.

Thus, if $Q'$ is the isogonal conjugate of $P'$ w.r.t. $\triangle ABC$, then $\{A,Q,Q'\}$ are colinear in this order. But $Q'\in\odot(AO)$ so $Q$ lies strictly inside $\odot(AO)$. Analogously, $Q$ lies strictly inside $\odot(BO),\odot(CO)$, which is a contradiction.

Remark: The lemma implies that the locus of $P$ which $A_2\in\odot(ABC)$ is a cubic curve.
Solution 2 (Nine-point conic) First, we prove the nine points conic (which is the generalization of nine-points circle) as follows.

Lemma: [Nine-points conic] The following nine points:
  • points $A_1,B_1,C_1$;
  • midpoints $M_A,M_B,M_C$ of $BC,CA,AB$; and
  • midpoints $A',B',C'$ of $AP,BP,CP$
lie on a conic.

Proof. Note parallel lines $M_AB'\parallel CP\parallel M_BA'$, $M_AC'\parallel M_CA'$, $M_AM_B\parallel AB\parallel A'B'$ and $M_AM_C\parallel A'C'$ so translating pencils gives
$$M_A(B',C';M_B,M_C) = A'(M_B,M_C;B',C') = A'(B',C';M_B,M_C)$$so $A',B',C',M_A,M_B,M_C$ lie on a conic. Finally by converse of Pascal's theorem on hexagon $M_AA_1A'C'B'M_C$ gives $A_1$ lie on this conic. Similarly $B_1,C_1$ lie on this conic. $\blacksquare$

Remark: The_Turtle pointed out above that this can be proven easily with affine transformation.

By taking homothety $\mathcal{H}(P,2)$, we get $A,B,C,A_2,B_2,C_2$ lie on a conic. Hence isogonal conjugate $A_2'$,$B_2'$,$C_2'$ of $A_2$,$B_2$,$C_2$ w.r.t. $\triangle ABC$ are colinear. However, if $A_2$ lie inside $\odot(ABC)$, one can check that $A_2'$ lies inside region determined by
  • extension of $BA$ beyond $A$, and
  • extension of $CA$ beyond $C$
that does not contain $\triangle ABC$. One can narrow positions of $B_2',C_2'$ similarly. This is enough to force that $A_2',B_2',C_2'$ are not colinear, contradiction.
This post has been edited 4 times. Last edited by MarkBcc168, Sep 27, 2020, 6:03 AM
Z K Y
The post below has been deleted. Click to close.
This post has been deleted. Click here to see post.
FISHMJ25
293 posts
#7 • 1 Y
Y by ImSh95
I have solution using moving points if someone can prove that if P satysfies that both $B_2,C_2$ lie on circumcircle then $P$ is orthocenter.
Z K Y
The post below has been deleted. Click to close.
This post has been deleted. Click here to see post.
GeoMetrix
924 posts
#8 • 2 Y
Y by Eliot, ImSh95
Ok this was ugly .

Solution
Z K Y
The post below has been deleted. Click to close.
This post has been deleted. Click here to see post.
miltosk
9 posts
#9 • 2 Y
Y by amar_04, ImSh95
I was just wondering if we can derive a contradiction from this:
If we prove that for whichever triangle that lies inside the circle (or on the boundaries) its perimeter is less than the double of the perimeter of the orthic triangle, we have a contradiction from Fagnano's problem.
I didnt check it though.
Z K Y
The post below has been deleted. Click to close.
This post has been deleted. Click here to see post.
OlyMan
124 posts
#10 • 2 Y
Y by ImSh95, Mango247
v_Enhance wrote:
Let $P = (x:y:z)$ with $x,y,z > 0$ so $A_1 = (0:y:z)$. Therefore, \begin{align*} 	A_2 &= \left( -x(y+z) : 2y(x+y+z) - (y+z)y : 2(x+y+z)-(y+z)z \right) \\ 	&= \left( -x(y+z) : y(2x+y+z) : z(2x+y+z)  \right). \end{align*}Claim: The point $A_2$ lies inside $(ABC)$ if and only if \[ 0 < a^2 \cdot yz(2x+y+z) - b^2 \cdot xz(y+z) - c^2xy(y+z). \]Proof. The condition we want is that \begin{align*} 	0 &< -\operatorname{Pow}(A_2, (ABC)) \\ 	&= a^2 \cdot y(2x+y+z) \cdot z(2x+y+z) \\ 		&\qquad - b^2x(y+z) \cdot z(2x+y+z) - c^2x(y+z) \cdot y(2x+y+z) \\ 	&= (2x+y+z) \left[ a^2 \cdot yz(2x+y+z) 		- b^2 \cdot xz(y+z) 		- c^2 \cdot xy(y+z) \right]. \end{align*}From $2x+y+z>0$ we conclude. $\blacksquare$
If all three points lie inside the circle, we sum cyclically now to get a contradiction.

Sir, what you have used? I don't even understand that. Do you use co-ordinate?
Z K Y
The post below has been deleted. Click to close.
This post has been deleted. Click here to see post.
mela_20-15
125 posts
#11 • 2 Y
Y by ImSh95, Ubfo
Consider the circumcevian triangle $A_3B_3C_3$ the problem states that not all three inequalities $A_1A_3>PA_1$ etc. can hold simultaneously.
Label the angles $\angle CAP=a_1$,$\angle PAB=a_2$,$\angle PBA=b_1$,$\angle CBP=b_2$ ,$\angle PCB=c_1$, $\angle ACP=c_2$ cyclically.
We have that $A_3BC=a_1$ and $A_3CB=a_2$ .
The following holds : $\frac{A_1A_3}{PA_1}=\frac{\frac{BC}{dist(P,BC)}}{\frac{BC}{dist(A_3,BC)}}=\frac{\cot{ b_2}+\cot{ c_1}}{\cot{a_1}+\cot{a_2}}>1$
Summing all three inequalities $  \cot{ b_2}+\cot{ c_1}>\cot{a_1}+\cot{a_2}$ we get a contradiction.
Z K Y
The post below has been deleted. Click to close.
This post has been deleted. Click here to see post.
Rg230403
222 posts
#13 • 10 Y
Y by Pluto1708, aops5234, MarkBcc168, Eliot, p_square, GeoMetrix, MathPassionForever, thczarif, jelena_ivanchic, ImSh95
Inside $\Delta ABC$, let a point be $X$, let $Y=AX\cap (ABC)$ and $Z$ is midpoint of $XY$. Let, isogonal conjugate of $X$ be $X'$ and of $Z$ be $Z'$. Let $Y'=AX'\cap (ABC)$. Now, we claim that
Midpoint of $Y'Z'$ is $X'$.
Proof: Let the point at infinity along $AX$ be $\infty_{AX}$ and along $AX'$ be $\infty_{AX'} $. Then, $-1=(XY;Z\infty_{AX})=(X'\infty_{AX'};Z'Y')$. This comes by isogonal conjugation.

So, we get the corollary that if the reflection of a point across $BC$ lies inside $(ABC)$ then the reflection of $A$ across its isogonal conjugate must lie inside $(ABC)$ as well. Thus, its isogonal conjugate must lie inside circle with diameter $AO$. But, no point $P$ can lie inside all 3 circles $(AO),(BO),(CO)$ and $\Delta ABC$ so we are done.

Note: I may add diagram later, too lazy.
This post has been edited 2 times. Last edited by Rg230403, Oct 1, 2020, 1:52 PM
Z K Y
The post below has been deleted. Click to close.
This post has been deleted. Click here to see post.
p_square
442 posts
#14 • 6 Y
Y by Aryan-23, MarkBcc168, Nathanisme, Rg230403, ImSh95, sabkx
There is/was actually a subtlety missed in the synthetic solutions in post #6 and post #13.

It is actually possible for a point $Q$ to lie inside all $3$ of the circles $\odot(AO), \odot(BO)$ and $\odot(CO)$!
[asy]
pair A = dir(110), B = dir(152), C = dir(28);
pair O = (0,0);
draw(A--B--C--cycle, royalblue);
draw(circumcircle(A,B,C),springgreen);
draw(circle((A+O)/2,0.5), fuchsia);
draw(circle((C+O)/2,0.5), fuchsia);
draw(circle((B+O)/2,0.5), fuchsia);
pair Q = (0.02, 0.2);
dot(Q, 3+orange);
label("$\underline{\underline{Q}}$",Q,dir(0),orange);

dot("$A$",A,dir(A),3+purple);
dot("$C$",C,dir(C),3+purple);
dot("$B$",B,dir(B),3+purple);
dot("$O$",O,dir(270),4+magenta);
[/asy]
The condition that $P$ lies inside triangle $ABC$ is actually necessary. The finish at the end, requires that no point $Q$ can lie inside $\odot(AO), \odot(BO), \odot(CO)$ and $\triangle ABC$. This could be shown via inversion around $\odot(ABC)$ or just by taking cases based on whether the triangle is a cute triangle.
Z K Y
The post below has been deleted. Click to close.
This post has been deleted. Click here to see post.
jj_ca888
2726 posts
#15 • 1 Y
Y by ImSh95
My hand was forced. Dam‍n this problem.

Let $AP \cap (ABC) = X$, $BP \cap (ABC) = Y$, $CP \cap (ABC) = Z$. Suppose otherwise FTSOC; we may assume that\[\left\{\frac{PA_1}{XA_1}, \frac{PB_1}{YB_1}, \frac{PC_1}{ZC_1}\right\}\]are all $< 1$. Notice that we can write $\tfrac{PA_1}{XA_1}$ in two ways; using ratio lemma on $\triangle  PBX$ and $\triangle PCX$:\[\frac{PA_1}{XA_1} = \frac{PB\sin{\angle PBC}}{XB\sin{\angle XAC}} = \frac{\sin{\angle C}\sin{\angle PBC}}{\sin{\angle APB}\sin{\angle PAC}} \]\[\frac{PA_1}{XA_1} = \frac{PC\sin{\angle PCB}}{XC\sin{\angle XAB}} = \frac{\sin{\angle B}\sin{\angle PCB}}{\sin{\angle APC}\sin{\angle PAB}} \]and similarly for the other two\[\frac{PB_1}{YB_1} = \frac{\sin{\angle A} \sin{\angle PCA}}{\sin{\angle BPC}\sin{\angle PBA}}\]\[\frac{PB_1}{YB_1} = \frac{\sin{\angle C} \sin{\angle PAC}}{\sin{\angle BPA}\sin{\angle PBC}}\]and\[\frac{PC_1}{ZC_1} = \frac{\sin{\angle B}\sin{\angle PAB}}{\sin{\angle BPA} \sin{\angle PCB}}\]\[\frac{PC_1}{ZC_1} = \frac{\sin{\angle A}\sin{\angle PBA}}{\sin{\angle APB} \sin{\angle PCA}}\]Notice that multiplying $(1), (4)$, then $(2), (5)$, and then $(3), (6)$, stuff nicely cancels out to yield\[\left(\frac{\sin{\angle C}}{\sin{\angle APB}}\right)^2, \left(\frac{\sin{\angle A}}{\sin{\angle BPC}}\right)^2, \left(\frac{\sin{\angle B}}{\sin{\angle CPA}}\right)^2 < 1\]which tells us that $\sin{\angle C} < \sin{\angle APB}$, $\sin{\angle A} < \sin{\angle BPC},$ and $\sin{\angle B} < \sin{\angle CPA}$. Since $P$ is inside the interior of $\triangle ABC$, we know that in fact $\angle C < \angle APB$, $\angle A < \angle BPC$, and $\angle B < \angle CPA$. Now, using sine subtraction formula with all three, we get\[\cos{(\tfrac{\angle APB + \angle C}{2})} \sin{(\tfrac{\angle APB - \angle C}{2})} > 0\]\[\cos{(\tfrac{\angle BPC + \angle A}{2})} \sin{(\tfrac{\angle BPC - \angle A}{2})} > 0\]\[\cos{(\tfrac{\angle CPA + \angle B}{2})} \sin{(\tfrac{\angle CPA - \angle B}{2})} > 0\]from which it follows that $\cos{(\tfrac{\angle APB + \angle C}{2})}, \cos{(\tfrac{\angle BPC + \angle A}{2})}, \cos{(\tfrac{\angle CPA + \angle B}{2})} > 0$. Since each of\[\angle APB + \angle C, \angle BPC + \angle A, \angle CPA + \angle B\]is bounded between $[0, 2\pi)$, it follows that each of them is $< \pi$ and summing yields $3\pi < 3\pi$, a contradiction, as desired. $\blacksquare$
This post has been edited 3 times. Last edited by jj_ca888, Oct 18, 2020, 10:31 PM
Z K Y
The post below has been deleted. Click to close.
This post has been deleted. Click here to see post.
TheUltimate123
1740 posts
#16 • 6 Y
Y by HamstPan38825, ImSh95, sabkx, Ru83n05, IAmTheHazard, Funcshun840
Solved with nukelauncher. Let \(\Gamma\) be the circumcircle. Consider an affine transformation that sends \(ABCP\) to an orthocentric system; then the images of \(A\), \(B\), \(C\), \(A_2\), \(B_2\), \(C_2\) lie on a circle. Undoing the affine transformation, points \(A\), \(B\), \(C\), \(A_2\), \(B_2\), \(C_2\) lie on an ellipse \(\mathcal E\).

[asy]         import olympiad;         import geometry;

pen pri=blue;         pen sec=purple+pink;         pen tri=lightblue;         pen qua=lightred;         pen fil=invisible;         pen tfil=invisible;         pen qfil=invisible;

size(7cm); defaultpen(fontsize(10pt));         pair A,B,C,P,A1,B1,C1,A2,B2,C2,D;         A=dir(110);         B=dir(210);         C=dir(330);         P=0.3*dir(250);         A1=extension(A,P,B,C);         B1=extension(B,P,C,A);         C1=extension(C,P,A,B);         A2=2A1-P;         B2=2B1-P;         C2=2C1-P;         path e=(path)conic(A,B,C,A2,B2);         for(int i=0;i<4;i+=1){             D=intersectionpoints(unitcircle,e)[i];             if(D==A||D==B||D==C)continue;             break;         }

filldraw(e,qfil,qua+dashed);         draw(A--A2,sec);         draw(B--B2,sec);         draw(C--C2,sec);         filldraw(unitcircle,tfil,tri);         filldraw(A--B--C--cycle,fil,pri);

dot("\(A\)",A,N);         dot("\(B\)",B,SW);         dot("\(C\)",C,SE);         dot("\(D\)",D,NE);         dot("\(P\)",P,dir(60));         dot("\(A_1\)",A1,SW);         dot("\(B_1\)",B1,dir(75));         dot("\(C_1\)",C1,dir(120));         dot("\(A_2\)",A2,S);         dot("\(B_2\)",B2,E);         dot("\(C_2\)",C2,W);     [/asy]

Without loss of generality let \(A\), \(B\), \(C\) lie on \(\Gamma\) in counterclockwise order. Evidently \(A_2\) lies on counterclockwise arc \(BC\) of \(\mathcal E\), and similarly for \(B_2\) and \(C_2\).

If \(\mathcal E=\Gamma\), then we are done. Otherwise \(\mathcal E\) and \(\Gamma\) intersect at one more point \(D\). Assume without loss of generality that \(D\) lies on counterclockwise arc \(CA\). Then either counterclockwise arc \(AB\) or \(BC\) of \(\mathcal E\) lies completely outside \(\Gamma\). Without loss of generality, arc \(AB\) lies outside; then \(C_2\) lies outside \(\Gamma\), as desired.
This post has been edited 1 time. Last edited by TheUltimate123, Jan 2, 2021, 11:06 AM
Z K Y
The post below has been deleted. Click to close.
This post has been deleted. Click here to see post.
BOBTHEGR8
272 posts
#17 • 1 Y
Y by ImSh95
Nice problem !
Solution
This post has been edited 2 times. Last edited by BOBTHEGR8, Mar 19, 2021, 10:33 AM
Reason: To correct error pointed by biomathematics
Z K Y
The post below has been deleted. Click to close.
This post has been deleted. Click here to see post.
guptaamitu1
656 posts
#18 • 2 Y
Y by ImSh95, sabkx
Another synthetic solution:

Assume for the sake of contradiction that points $A_1,B_1,C_1$ all lie strictly inside the circumcircle of $\triangle ABC$.


Claim 1: If $\angle AA_1B \le 90^\circ$, then $\angle BAP > \angle BCP$ and other analogous results.

proof: Suppose $\angle AA_1B \le 90^\circ$. Let line $AP$ meet $\odot(ABC)$ again at $A_3$. Then by our assumption, $PA_1 < PA_3$. Let $A_3'$ be the reflection of $A_3$ in line $BC$. Then $\angle BAP = \angle BAA_3 = \angle BCA_3 = \angle BCA_3'$. So it suffices to show $\angle BCA_3' > \angle BCP$. Let the line through $A_3'$ parallel to $\overline{BC}$ cut line $AP$ at $P'$. Then by midpoint theorem $A_1P' = A_1A3 > A_1P$, so $P$ strictly lies between $A_1,P'$ so $\angle BCP' > \angle BCP$. Also, and $\angle AA_1B \le 90^\circ$, so because of the configuration we must have that $\angle BCA_3' \ge \angle BCP'$. Hence $\angle BCA_3' > \angle BCP$. This proves the claim. $\square$
[asy]
pair A=dir(130),B=dir(-150),C=dir(-30),A3=dir(-85),A1=extension(A,A3,B,C),P=1.6*A1-0.6*A3,A3p=2*foot(A3,B,C)-A3,Pp=2*A1-A3,A0=foot(A3,B,C);
draw(unitcircle,brown);
dot("$A$",A,dir(A));
dot("$B$",B,dir(B));
dot("$C$",C,dir(C));
dot("$A_1$",A1,dir(A1));
dot("$A_3$",A3,dir(-40));
dot("$P$",P,dir(P));
dot("$P'$",Pp,dir(160));
dot("$A_3'$",A3p,dir(-50));
dot("$A_0$",A0,dir(-45));
draw(B--A--C,magenta);
draw(B--C^^Pp--A3p,red);
draw(A--A3^^A3--A3p,green);
[/asy]
Corollary: If $\angle PA_1B \le 90^\circ$ then $\angle PC_1B > 90^\circ$.

proof: Easy proof by contradiction. $\square$


Now assume WLOG that $\angle AA_1B \le 90^\circ$. Then using our Corollary we obtain
\begin{align*}
& \angle PC_1B > 90^\circ ~ \implies ~ \angle PC_1A < 90^\circ ~ \implies ~ \angle PB_1A > 90^\circ ~ \\
&  \implies ~ \angle PB_1C < 90^\circ ~ \implies \angle PA_1C > 90^\circ ~ \implies \angle PA_1B < 90^\circ 
\end{align*}In particular, we have that
$$\angle PA_1B , \angle PC_1A , \angle PB_1C < 90^\circ ~,~ \angle PA_1C , \angle PC_1B, \angle PB_1A > 90^\circ $$Now let $H$ be the orthocenter of $\triangle ABC$ and $D,E,F$ as usual. The conditions we obtain imply that $P$ lies strictly inside each of the following triangles
$$\triangle ADC ~,~ \triangle BEA ~,~ \triangle CFB$$But it is easy to see that these three triangle do not have a common interior point. So we have a contradiction!
This completes the proof of the problem. $\blacksquare$
Z K Y
The post below has been deleted. Click to close.
This post has been deleted. Click here to see post.
Mahdi_Mashayekhi
689 posts
#19 • 1 Y
Y by ImSh95
Interesting
Let $AP,BP,CP$ meet $ABC$ at $Y,Z,X$. Note that problem actually means at least one of $\frac{PA_1}{A_1Y},\frac{PB_1}{B_1Z},\frac{PC_1}{C_1X} \ge 1$ so Assume not.
Note that $\frac{PA_1}{A_1Y} = \frac{\frac{PA_1}{A_1B}}{\frac{A_1Y}{A_1B}} = \frac{\frac{PA_1}{A_1C}}{\frac{A_1Y}{A_1C}}$ so $\frac{\frac{\sin{PBC}}{\sin{BPA}}}{\frac{\sin{PAC}}{\sin{BCA}}} = \frac{\frac{\sin{PCB}}{\sin{CPA}}}{\frac{\sin{PAB}}{\sin{CBA}}}$. we do same approach for $\frac{PB_1}{B_1Z},\frac{PC_1}{C_1X}$. Now we have $(\frac{\sin BAC}{\sin BPC})^2 = \frac{PB_1}{B_1Z}\frac{PC_1}{C_1X} < 1$. we have same approach for $(\frac{\sin BCA}{\sin BPA})^2$ and $(\frac{\sin ABC}{\sin APC})^2$ but then we should have $\angle BAC < \angle 180 - \angle BPC$ and $\angle BCA < \angle 180 - \angle BPA$ and $\angle ABC < \angle 180 - \angle APC$ which is obviously not true so contradiction so at least one of $\frac{PA_1}{A_1Y},\frac{PB_1}{B_1Z},\frac{PC_1}{C_1X} \ge 1$ so at least one of $A_2,B_2,C_2$ lies outside or on $ABC$ itself.
we're Done.
Z K Y
The post below has been deleted. Click to close.
This post has been deleted. Click here to see post.
lrjr24
966 posts
#20
Y by
We use barycentric coordinates with respect to $ABC$. Let $P=(p:q:r)$. We have that $A_1=(0:\frac{p+q+r}{q+r} \cdot q : \frac{p+q+r}{q+r})$. We have that $A_2=(-p:q \cdot \frac{2p+q+r}{q+r}: r \cdot \frac{2p+q+r}{q+r})$. Note that $A_2$ lying inside of the circumcircle of $ABC$ is equivalent to $$0 > \text{Pow}((ABC),A_2) \iff a^2qr\left( \frac{2p+q+r}{q+r} \right) ^2-b^2pr \cdot \frac{2p+q+r}{q+r}-c^2pq \cdot \frac{2p+q+r}{q+r} > 0 \iff a^2qr(2p+q+r)>b^2pr(q+r)+c^2pq(q+r).$$If all the points lie inside the circumcircle of $ABC$, we get a contradiction by summing cyclically we get a contradiction.
This post has been edited 1 time. Last edited by lrjr24, Aug 10, 2022, 4:41 PM
Z K Y
The post below has been deleted. Click to close.
This post has been deleted. Click here to see post.
JAnatolGT_00
559 posts
#21 • 1 Y
Y by Mango247
There exist an affine transformation which maps $A,B,C,P$ onto orthocentric quadruple, and so maps $$\left \{ A,B,C,A_2,B_2,C_2 \right \} \quad (\star )$$onto set of concyclic points. This yields that points from $(\star )$ lie on one ellipse $\varepsilon.$

Case $\varepsilon =\odot (ABC)$ is obvious, so assume the opposite. Note that $A,C_2,B,A_2,C,A_2$ lie on $\varepsilon$ in that order; WLOG arc $AC_2$ of ellipse intersects $\odot (ABC)$ again at $D.$ One of arc $AC,BC$ of $\varepsilon$ lies outside $\odot (ABC),$ hence either $B_2$ or $A_2$ can't lie strictly inside the circumcircle, as required.
This post has been edited 1 time. Last edited by JAnatolGT_00, Jan 3, 2023, 9:29 PM
Z K Y
The post below has been deleted. Click to close.
This post has been deleted. Click here to see post.
awesomeming327.
1677 posts
#22 • 1 Y
Y by Inconsistent
Let $A_3$, $B_3$, $C_3$ be the intersections of $AP$, $BP$, $CP$ on $(ABC)$. Let $\theta_1=\angle ABB_3$, $\theta_2=\angle CBB_3$, $\theta_3=\angle CAA_3$, $\theta_4=\angle BAA_3$, $\theta_5=\angle BCC_3$, and $\theta_6=\angle ACC_3$. We have
\begin{align*}
\frac{C_1C_3}{PC_1} &= \frac{\sin(\theta_3+\theta_6)\sin(\theta_5)}{\sin(\theta_1+\theta_2)\sin(\theta_4)} = \frac{\sin(\theta_2+\theta_5)\sin(\theta_6)}{\sin(\theta_3+\theta_4)\sin(\theta_1)}\\
\frac{B_1B_3}{PB_1} &= \frac{\sin(\theta_2+\theta_5)\sin(\theta_1)}{\sin(\theta_3+\theta_4)\sin(\theta_6)} = \frac{\sin(\theta_1+\theta_4)\sin(\theta_2)}{\sin(\theta_5+\theta_6)\sin(\theta_3)}\\
\frac{A_1A_3}{PA_1} &= \frac{\sin(\theta_1+\theta_4)\sin(\theta_3)}{\sin(\theta_5+\theta_6)\sin(\theta_2)} = \frac{\sin(\theta_3+\theta_6)\sin(\theta_4)}{\sin(\theta_1+\theta_2)\sin(\theta_5)}
\end{align*}Suppose all those are less than one, then the following are also less than one:
\begin{align*}
\sqrt{\frac{C_1C_3}{PC_1} \cdot \frac{B_1B_3}{PB_1}} &= \sqrt{\frac{\sin(\theta_2+\theta_5)\sin(\theta_6)}{\sin(\theta_3+\theta_4)\sin(\theta_1)}\cdot \frac{\sin(\theta_2+\theta_5)\sin(\theta_1)}{\sin(\theta_3+\theta_4)\sin(\theta_6)}} \\
&= \frac{\sin(\theta_2+\theta_5)}{\sin(\theta_3+\theta_4)} \\
\sqrt{\frac{B_1B_3}{PB_1} \cdot \frac{A_1A_3}{PA_1}} &= 
\sqrt{\frac{\sin(\theta_1+\theta_4)\sin(\theta_2)}{\sin(\theta_5+\theta_6)\sin(\theta_3)}\cdot \frac{\sin(\theta_1+\theta_4)\sin(\theta_3)}{\sin(\theta_5+\theta_6)\sin(\theta_2)}} \\
&= \frac{\sin(\theta_1+\theta_4)}{\sin(\theta_5+\theta_6)} \\
\sqrt{\frac{A_1A_3}{PA_1} \cdot \frac{C_1C_3}{PC_1}} &= 
\sqrt{\frac{\sin(\theta_3+\theta_6)\sin(\theta_4)}{\sin(\theta_1+\theta_2)\sin(\theta_5)}\cdot \frac{\sin(\theta_3+\theta_6)\sin(\theta_5)}{\sin(\theta_1+\theta_2)\sin(\theta_4)}} \\
&= \frac{\sin(\theta_3+\theta_6)}{\sin(\theta_1+\theta_2)}
\end{align*}Therefore,
\begin{align*}
\theta_2+\theta_5&<\theta_3+\theta_4  \\
\theta_1+\theta_4&<\theta_5+\theta_6 \\
\theta_3+\theta_6&<\theta_1+\theta_2
\end{align*}Which is absurd.
Z K Y
The post below has been deleted. Click to close.
This post has been deleted. Click here to see post.
awesomeming327.
1677 posts
#23 • 1 Y
Y by Inconsistent
And I have one comment: ew.
Z K Y
The post below has been deleted. Click to close.
This post has been deleted. Click here to see post.
Leo.Euler
577 posts
#24 • 1 Y
Y by Inconsistent
Thanks to pi_is_3.14 for quoting affine transformations numerous times at HMMT (which led me to reading about it!).

The key idea is that there exists an affine transformation $\varphi$ mapping $A, B, C, P$ to an orthocentric system; note that $\varphi$ also maps $\{ A, B, C, A_2, B_2, C_2 \}$ to a set of concyclic points by reflecting the orthocenter. Taking $\varphi^{-1}$ of the transformed image yields that $A, B, C, A_2, B_2, C_2$ all lie on an ellipse.

Case 1: $\{A_2, B_2, C_2\} \subseteq (ABC)$
This case trivially works.
Case 2: $\{A_2, B_2, C_2\} \not\subseteq (ABC)$
Note that the ellipse containing $A, B, C, A_2, B_2, C_2$ and $(ABC)$ intersect at one more point, say $D$. WLOG suppose that $D$ lies on the arc $CA$ not containing point $B$. Then one of arcs $AC$ and $BC$ of the ellipse lies outside of $(ABC)$, so either $B_2$ or $A_2$ do not lie strictly inside $(ABC)$, and we are done.
This post has been edited 1 time. Last edited by Leo.Euler, Jun 15, 2023, 5:01 AM
Z K Y
The post below has been deleted. Click to close.
This post has been deleted. Click here to see post.
KevinYang2.71
410 posts
#25 • 1 Y
Y by deduck
We use barycentric coordinates with $A=(1,0,0)$, $B=(0,1,0)$, and $C=(0,0,1)$. Let $P=:(p,q,r)$ so
\begin{align*}
A_1&=\left(0,\frac{q}{1-p},\frac{r}{1-p}\right)\\
B_1&=\left(\frac{p}{1-q},0,\frac{r}{1-q}\right)\\
B_1&=\left(\frac{p}{1-r},\frac{q}{1-r},0\right).
\end{align*}Then
\begin{align*}
A_2&=2A_1-P&=\left(-p,\frac{pq+q}{1-p},\frac{pr+r}{1-p}\right)\\
B_2&=2B_1-P&=\left(\frac{qp+p}{1-p},-q,\frac{qr+r}{1-p}\right)\\
C_2&=2C_1-P&=\left(\frac{rp+p}{1-p},\frac{rq+q}{1-p},-r\right)
\end{align*}so
\begin{align*}
\mathrm{Pow}_{(ABC)}(A_2)&=-a^2\cdot\frac{pq+q}{1-p}\cdot\frac{pr+r}{1-p}+b^2p\cdot\frac{pr+r}{1-p}+c^2p\cdot\frac{pq+q}{1-p}\\
&=-\frac{p+1}{(1-p)^2}\left(a^2qr(p+1)+b^2rp(p-1)+c^2pq(p-1)\right).
\end{align*}Similarly,
\begin{align*}
\mathrm{Pow}_{(ABC)}(B_2)&=-\frac{q+1}{(1-q)^2}\left(a^2qr(q-1)+b^2rp(q+1)+c^2pq(q-1)\right)\\
\mathrm{Pow}_{(ABC)}(C_2)&=-\frac{r+1}{(1-r)^2}\left(a^2qr(r-1)+b^2rp(r-1)+c^2pq(r+1)\right)
\end{align*}so
\[
\frac{(1-p)^2}{p+1}\mathrm{Pow}_{(ABC)}(A_2)+\frac{(1-q)^2}{q+1}\mathrm{Pow}_{(ABC)}(B_2)+\frac{(1-r)^2}{r+1}\mathrm{Pow}_{(ABC)}(C_2)=0
\](note that $p+q+r-1=0$). Since $p,q,r>0$ and thus $\frac{(1-p)^2}{p+1},\frac{(1-q)^2}{q+1},\frac{(1-r)^2}{r+1}>0$, it follows that at least one of $\mathrm{Pow}_{(ABC)}(A_2),\mathrm{Pow}_{(ABC)}(B_2),\mathrm{Pow}_{(ABC)}(C_2)$ must be positive. Thus at least one of $A_2,B_2,C_2$ is not inside $(ABC)$. $\square$
Z K Y
The post below has been deleted. Click to close.
This post has been deleted. Click here to see post.
atdaotlohbh
171 posts
#26
Y by
My ugly solution for acute-angled triangle :-D
FTSOC, suppose $A_2,B_2,C_2$ all lie strictly inside, then $BA_1*A_1C>A_1P*A_1A$ and its cyclic versions. Call $F_A=\frac{BA_1*A_1C}{A_1P*A_1A}$
By writing Law of Sines for $BC_1P, PB_1C, ABB_1,AC_1C$ we get
$\frac{F_B}{F_C}=(\frac{sin\angle ABB_1}{sin\angle ACC_1})^2$
Now WLOG suppose $F_C$ is the smallest among the three. Then from the condition above $\angle ABP \geq \angle PCA$ and $\angle BAP \geq \angle PCB$.
Let $M$ be the intersection of $CP$ with circumcircle of $ABC$, Then $C_1P<C_1M$, $\angle PAC_1\geq \angle MAC_1$, $\angle PBC_1 \geq \angle C_1BM$, from which $AP<AM$ and $BP<BM$. But then $\angle AC_1P=\angle AMC_1+\angle MAC_1<\angle C_1AP+\angle C_1PA=\angle BC_1P$ and similarly $\angle BC_1P<\angle AC_1P$, a contradiction
This post has been edited 2 times. Last edited by atdaotlohbh, Jun 8, 2024, 7:37 PM
Z K Y
The post below has been deleted. Click to close.
This post has been deleted. Click here to see post.
Scilyse
386 posts
#27 • 1 Y
Y by ohiorizzler1434
blame Angelo Di Pasquale and Andrew Elvey Price for this problem
Z K Y
N Quick Reply
G
H
=
a